Summer is a great time to explore cool problems to keep your skills sharp!  Schedule a class today!

G
Topic
First Poster
Last Poster
You would not believe your eyes...
willwin4sure   12
N an hour ago by pi271828
Source: USA TSTST 2020 Problem 9, by Nikolai Beluhov
Ten million fireflies are glowing in $\mathbb{R}^3$ at midnight. Some of the fireflies are friends, and friendship is always mutual. Every second, one firefly moves to a new position so that its distance from each one of its friends is the same as it was before moving. This is the only way that the fireflies ever change their positions. No two fireflies may ever occupy the same point.

Initially, no two fireflies, friends or not, are more than a meter away. Following some finite number of seconds, all fireflies find themselves at least ten million meters away from their original positions. Given this information, find the greatest possible number of friendships between the fireflies.

Nikolai Beluhov
12 replies
willwin4sure
Jan 25, 2021
pi271828
an hour ago
Rectangle EFGH in incircle, prove that QIM = 90
v_Enhance   65
N an hour ago by zuat.e
Source: Taiwan 2014 TST1, Problem 3
Let $ABC$ be a triangle with incenter $I$, and suppose the incircle is tangent to $CA$ and $AB$ at $E$ and $F$. Denote by $G$ and $H$ the reflections of $E$ and $F$ over $I$. Let $Q$ be the intersection of $BC$ with $GH$, and let $M$ be the midpoint of $BC$. Prove that $IQ$ and $IM$ are perpendicular.
65 replies
v_Enhance
Jul 18, 2014
zuat.e
an hour ago
2-var inequality
sqing   7
N an hour ago by sqing
Source: Own
Let $ a,b\geq 0 $ and $\frac{1}{a^2+3} + \frac{1}{b^2+3} -ab\leq  \frac{1}{2}.$ Prove that
$$  a^2+ab+b^2 \geq \frac{3(\sqrt{57}-7)}{4}$$Let $ a,b\geq 0 $ and $\frac{a}{b^2+3} + \frac{b}{a^2+3} +ab\leq  \frac{1}{2}.$ Prove that
$$  a^2+ab+b^2 \leq \frac{9}{4}$$Let $ a,b\geq 0 $ and $ \frac{a}{b^3+3}+\frac{b}{a^3+3}-ab\leq  \frac{1}{2}.$ Prove that
$$  a^2+ab+b^2 \geq \frac{9}{4}$$
7 replies
sqing
Yesterday at 12:55 PM
sqing
an hour ago
Number theory
falantrng   40
N an hour ago by bjump
Source: RMM 2018 D2 P4
Let $a,b,c,d$ be positive integers such that $ad \neq bc$ and $gcd(a,b,c,d)=1$. Let $S$ be the set of values attained by $\gcd(an+b,cn+d)$ as $n$ runs through the positive integers. Show that $S$ is the set of all positive divisors of some positive integer.
40 replies
falantrng
Feb 25, 2018
bjump
an hour ago
Calvin needs to cover all squares
Rijul saini   4
N 2 hours ago by SimplisticFormulas
Source: India IMOTC 2025 Day 2 Problem 1
Consider a $2025\times 2025$ board where we identify the squares with pairs $(i,j)$ where $i$ and $j$ denote the row and column number of that square, respectively.

Calvin picks two positive integers $a,b<2025$ and places a pawn at the bottom left corner (i.e. on $(1,1)$) and makes the following moves. In his $k$-th move, he moves the pawn from $(i,j)$ to either $(i+a,j)$ or $(i,j+a)$ if $k$ is odd and to either $(i+b,j)$ and $(i,j+b)$ if $k$ is even. Here all the numbers are taken modulo $2025$. Find the number of pairs $(a,b)$ that Calvin could have picked such that he can make moves so that the pawn covers all the squares on the board without being on any square twice.

Proposed by Tejaswi Navilarekallu
4 replies
Rijul saini
Yesterday at 6:35 PM
SimplisticFormulas
2 hours ago
Different scores possible in interview
Rijul saini   3
N 2 hours ago by Adywastaken
Source: India IMOTC Practice Test 2 Problem 1
In a job interview, the candidates are asked questions in a sequence. The initial score is $0$. The candidate's score is calculated as follows:

$\bullet$ after a correct answer, the score is increased by $1$;
$\bullet$ after a wrong answer, the score is divided by $2$.

If the candidate is asked $n$ questions and answers all of them, how many different scores are possible?

Note: Two different response sequences of the same length can result in the same score: the sequences $RRW$ and $WWR$ with the same length, where $R$ denotes the correct answer and $W$ denotes the wrong answer, both result in the same score of 1.

Proposed by S. Muralidharan
3 replies
Rijul saini
Yesterday at 6:54 PM
Adywastaken
2 hours ago
Concurrence
LiamChen   0
2 hours ago
Source: MOP1998
Problem:
0 replies
LiamChen
2 hours ago
0 replies
set of points, there exist two lines containing n points
jasperE3   1
N 2 hours ago by ririgggg
Source: 2004 Brazil TST Test 2 P1
Find the smallest positive integer $n$ that satisfies the following condition: For every finite set of points on the plane, if for any $n$ points from this set there exist two lines containing all the $n$ points, then there exist two lines containing all points from the set.
1 reply
jasperE3
Apr 5, 2021
ririgggg
2 hours ago
XY is tangent to a fixed circle
a_507_bc   2
N 2 hours ago by math-olympiad-clown
Source: Baltic Way 2022/15
Let $\Omega$ be a circle, and $B, C$ are two fixed points on $\Omega$. Given a third point $A$ on $\Omega$, let $X$ and $Y$ denote the feet of the altitudes from $B$ and $C$, respectively, in the triangle $ABC$. Prove that there exists a fixed circle $\Gamma$ such that $XY$ is tangent to $\Gamma$ regardless of the choice of the point $A$.
2 replies
a_507_bc
Nov 12, 2022
math-olympiad-clown
2 hours ago
Super easy problem
M11100111001Y1R   6
N 2 hours ago by sami1618
Source: Iran TST 2025 Test 2 Problem 1
The numbers from 2 to 99 are written on a board. At each step, one of the following operations is performed:

$a)$ Choose a natural number \( i \) such that \( 2 \leq i \leq 89 \). If both numbers \( i \) and \( i+10 \) are on the board, erase both.

$b)$ Choose a natural number \( i \) such that \( 2 \leq i \leq 98 \). If both numbers \( i \) and \( i+1 \) are on the board, erase both.

By performing these operations, what is the maximum number of numbers that can be erased from the board?
6 replies
M11100111001Y1R
May 27, 2025
sami1618
2 hours ago
Medium geometry with AH diameter circle
v_Enhance   95
N May 11, 2025 by Markas
Source: USA TSTST 2016 Problem 2, by Evan Chen
Let $ABC$ be a scalene triangle with orthocenter $H$ and circumcenter $O$. Denote by $M$, $N$ the midpoints of $\overline{AH}$, $\overline{BC}$. Suppose the circle $\gamma$ with diameter $\overline{AH}$ meets the circumcircle of $ABC$ at $G \neq A$, and meets line $AN$ at a point $Q \neq A$. The tangent to $\gamma$ at $G$ meets line $OM$ at $P$. Show that the circumcircles of $\triangle GNQ$ and $\triangle MBC$ intersect at a point $T$ on $\overline{PN}$.

Proposed by Evan Chen
95 replies
v_Enhance
Jun 28, 2016
Markas
May 11, 2025
Medium geometry with AH diameter circle
G H J
Source: USA TSTST 2016 Problem 2, by Evan Chen
The post below has been deleted. Click to close.
This post has been deleted. Click here to see post.
v_Enhance
6882 posts
#1 • 25 Y
Y by doxuanlong15052000, john111111, anantmudgal09, Davi-8191, AlastorMoody, Vietjung, rashah76, itslumi, v4913, tenebrine, sotpidot, HWenslawski, innout, anonman, megarnie, HamstPan38825, Jc426, Lamboreghini, CyclicISLscelesTrapezoid, tiendung2006, mathmax12, Adventure10, Mango247, geobo, Rounak_iitr
Let $ABC$ be a scalene triangle with orthocenter $H$ and circumcenter $O$. Denote by $M$, $N$ the midpoints of $\overline{AH}$, $\overline{BC}$. Suppose the circle $\gamma$ with diameter $\overline{AH}$ meets the circumcircle of $ABC$ at $G \neq A$, and meets line $AN$ at a point $Q \neq A$. The tangent to $\gamma$ at $G$ meets line $OM$ at $P$. Show that the circumcircles of $\triangle GNQ$ and $\triangle MBC$ intersect at a point $T$ on $\overline{PN}$.

Proposed by Evan Chen
Z K Y
The post below has been deleted. Click to close.
This post has been deleted. Click here to see post.
v_Enhance
6882 posts
#2 • 25 Y
Y by anantmudgal09, jlammy, math4444, baladin, Davi-8191, Iyerie, Anar24, AlastorMoody, Omeredip, amar_04, mathlogician, v4913, tenebrine, sotpidot, TheCollatzConjecture, innout, megarnie, hsiangshen, HamstPan38825, Jc426, Lamboreghini, mathmax12, EpicBird08, Adventure10, Mango247
Denote by $\triangle DEF$ the orthic triangle. Note that $\overline{AG}$, $\overline{EF}$, $\overline{BC}$ are concurrent at $R$ by radical axis, and that $\overline{PA}$ and $\overline{PG}$ are tangents to $\gamma$.

Now, consider circles $(PAGM)$, $(MFDNE)$, and $(MBC)$. They intersect at $M$ but have radical center $R$, so are coaxial; assume they meet again at $T \in \overline{RM}$, say. Then $\angle PTM$ and $\angle MTN$ are both right angles, hence $T$ lies on $\overline{PN}$.

Finally $H$ is the orthocenter of $\triangle ARN$, and thus the circle with diameter $\overline{RN}$ passes through $G$, $Q$, $N$.
This post has been edited 2 times. Last edited by v_Enhance, Jun 29, 2016, 6:48 AM
Reason: Un-hide now that people have had chance to try it
Z K Y
The post below has been deleted. Click to close.
This post has been deleted. Click here to see post.
LeVietAn
375 posts
#3 • 3 Y
Y by AllanTian, Adventure10, Mango247
My solutions:
We have $\overline{G, H, N}$ (well-known). Let $AG\cap BC=S, AH\cap BC=D$, and let $MK\perp PN at K$.
We have $AG, KM, DN$ are concurent at point $S$ is the radical center of ${\odot (APGKM), \odot (MKDN), \odot (AGDN)}$.
Becaues $AH\perp SN, NH\perp AS$ $\Rightarrow H$ is othorcenter of $\triangle ANS$ $\Rightarrow Q\in SH$.
We have $SK.SM=SG.SA=SB.SC$ $\Rightarrow K\in \odot (BCM)$.
We have $K\in \odot (SN)\equiv (GQN)$.
So, $(GNQ)\cap (MBC)\cap PN=K$. DONE
Z K Y
The post below has been deleted. Click to close.
This post has been deleted. Click here to see post.
ABCDE
1963 posts
#4 • 4 Y
Y by char2539, amuthup, Adventure10, Mango247
I always overcomplicate Evan's medium geometry proposals lol

Click to reveal hidden text
Z K Y
The post below has been deleted. Click to close.
This post has been deleted. Click here to see post.
houssam9990
33 posts
#6 • 2 Y
Y by Adventure10, Mango247
Click to reveal hidden text
This post has been edited 1 time. Last edited by houssam9990, Jun 28, 2016, 4:21 PM
Z K Y
The post below has been deleted. Click to close.
This post has been deleted. Click here to see post.
ThE-dArK-lOrD
4071 posts
#7 • 3 Y
Y by talkon, Adventure10, Mango247
Let $U,V$ be the feet of altitudes from $B,C$ respectively.
Let $UV\cap BC=X$ and let $XM \cap \mathcal{N} =T$ where $\mathcal{N}$ is the nine-point circle of $\triangle{ABC}$.
Considering radical axes of $\mathcal{N},(ABC),(MBC)$ we found that $X$ is the radical center and so $T \in (MBC)$.
Considering those of $(PAG),(ABC),(MBC)$ we found that $X$ is the radical center and so $T\in (PAG)$.
Then, we have $\angle{PTM}=90^{\circ}$ and $\angle{NTM}=\angle{NUM}=90^{\circ}$. So $T\in PN$.
From $\angle{XTN}=90^{\circ}=\angle{XQN}=\angle{XGN}=90^{\circ}$ we get $T\in (GQN)$.
Hence $(GQN)\cap(MBC)=T\in PN$.
This post has been edited 4 times. Last edited by ThE-dArK-lOrD, Jun 13, 2019, 6:04 AM
Z K Y
The post below has been deleted. Click to close.
This post has been deleted. Click here to see post.
talkon
276 posts
#8 • 3 Y
Y by ThE-dArK-lOrD, Adventure10, Mango247
My solution.
This post has been edited 3 times. Last edited by talkon, Jun 28, 2016, 5:36 PM
Z K Y
The post below has been deleted. Click to close.
This post has been deleted. Click here to see post.
liberator
95 posts
#9 • 13 Y
Y by abk2015, v_Enhance, PARISsaintGERMAIN, BobaFett101, JasperL, rashah76, parola, Abidabi, sotpidot, ike.chen, Adventure10, Mango247, Rounak_iitr
[asy]
unitsize(3.5cm);
pointpen=black; pathpen=rgb(0.4,0.6,0.8);
pointfontpen=fontsize(10);

path carc(pair A, pair B, pair C, real d=0, bool dir=CW) {
pair O=circumcenter(A,B,C);
return arc(O,circumradius(A,B,C),degrees(A-O)+d,degrees(C-O)-d,dir);
}

pair A=dir(120), B=dir(205), C=dir(-25), E=foot(B,C,A), F=foot(C,A,B), H=A+B+C, O=(0,0), M=(A+H)/2, N=(B+C)/2, R=extension(E,F,B,C), G=foot(N,A,R), P=extension(O,M,A,A+C-B), Q=extension(R,H,A,N), T=foot(M,P,N);

D(unitcircle,heavygreen);
DPA(CP(M,A)^^A--P--G,red);
D(carc(R,Q,N),purple);
D(carc(B,M,C),orange);
D(A--B--C--cycle,1);
DPA(A--R--E^^B--R--M^^P--N);
DPA(H--A--N^^P--O,dashed+pathpen);

D("A",A,dir(A));
D("B",B,SW);
D("C",C,SE);
D("E",E,dir(0));
D("F",F,dir(F));
D("G",G,NW);
D("H",H);
D("M",M,NE);
D("N",N);
D("O",O);
D("P",P,dir(P));
D("Q",Q);
D("R",R,dir(R));
D("T",T);
[/asy]
Let $\triangle DEF$ be the orthic triangle, and note that $AG,EF,BC$ are concurrent at $R$ by radical axis. Let $T=MR\cap PN$. With respect to $\gamma$, $\overline{AG}$ is the polar of $P$ and $\overline{EF}$ is the polar of $N$, thus $\overline{PN}$ is the polar of $R$ (La Hire), so $R,T$ are inverses, and $\angle RTN=90^{\circ}$.

$H$ is the orthocenter of $\triangle ARN$ (Brokard), and thus the circle on diameter $\overline{RN}$ passes through $G,T,Q,N$. But also $RT\cdot RM=RE\cdot RF=RB\cdot RC$, so $T$ lies on $(MBC)$, and the result follows.

Comment. This is a nice, novel problem based off a popular configuration. Well done Evan!
This post has been edited 1 time. Last edited by liberator, Jun 28, 2016, 5:37 PM
Reason: added comment
Z K Y
The post below has been deleted. Click to close.
This post has been deleted. Click here to see post.
EulerMacaroni
851 posts
#10 • 2 Y
Y by Adventure10, Mango247
Let $DEF$ be the orthic triangle, $V$ be the harmonic conjugate of $D$ with respect to segment $\overline{BC}$, and $T'$ be the harmonic conjugate of $D$ with respect to $EF$ in the nine-point circle. Projecting from $M$ onto line $EF$ implies that $M,T',V$ are collinear, so $$VT'\cdot VM=VF\cdot VE=VB\cdot VC$$so $T'\in\odot(MBC)$, and since $\angle MT'N=\pi-\angle VT'N=\frac{\pi}{2}$, $T'\in \odot(GQN)$, hence $T'\equiv T$. Finally, $$VT\cdot VM=VB\cdot VC=VG\cdot VA$$so pentagon $APGTM$ is cyclic, and $\angle PTM=\pi-\angle PAM=\frac{\pi}{2}$.
Z K Y
The post below has been deleted. Click to close.
This post has been deleted. Click here to see post.
First
2352 posts
#11 • 1 Y
Y by Adventure10
V_Enhance, how did you come up with this problem?
Z K Y
The post below has been deleted. Click to close.
This post has been deleted. Click here to see post.
PROF65
2016 posts
#12 • 2 Y
Y by Adventure10, Mango247
Let $ S= AG\cap BC $ we know that $NG\cap SQ=H$ is the orthocenter of $ASN$ if J is the center of $NSGQ$ then $\widehat{JGM}=\widehat{GNS}=\widehat{GQS}=\widehat{GQH} $ thus $GJ$ is tangent to $(M)$ i.e. $\overline{JGP} \perp GM $ besides $   OM \perp AG,NG\perp AG $ then $OM \parallel NG$ implies $ \widehat{GPM}=\widehat{JGN}=\widehat{GQH}=\widehat{GAH}    =\widehat{GAM} $ hence $GPMA$ s cyclic but $GJ\perp GM$ thus $PA \perp AM=AH$ implies $PA\parallel BC $ , applying the converse of Reim's to $N-R-P , S-G-A$ where $ R=NP\cap (GMAP) $ yields $ GRSB$ is cyclic .In the other hand $RM\perp PR, SR\perp NR $ we deduce $S,R,M $ are collinear ; further $GA,RM,BC$ are concurrent so by radical axis 's we conclude $RMBC$ is cyclic.
R HAS.
Z K Y
The post below has been deleted. Click to close.
This post has been deleted. Click here to see post.
DrMath
2130 posts
#13 • 2 Y
Y by Adventure10, Mango247
Nice problem!

Let $\triangle DEF$ be the orthic triangle. First, $\angle AQH=\angle AQA' = 90$ where $A'$ is the antipode of $A$. Thus $Q, H, N, A'$ are collinear. An easy angle chase gives $\angle MFN =\angle MEN =90$, so $QH$, the tangent to $(AEF)$ at $E$, and the tangent at $F$ concur. Thus $QFHE$ is harmonic. A perspectivity at $A$ gives $XBDC$ harmonic where $X=AQ\cap BC$. Thus $X\in EF$ by a well known lemma. On the other hand, since $NH\perp AQ$ and $AH\perp XN$, $XH\perp AG$ and $X, H, G$ are collinear. Moreover, $(XQGN)$ is cyclic.

Let $T$ be the intersection of circles $(XQGN)$ and $(MFDNE)$ other than $N$. Note $XT\cdot XM = XF\cdot XE=XB\cdot XC$, so $T$ lies on $(MBC)$. By Radical Axis on $(AQFHGE), (XQGN), (MFDNE)$, $GQ, EF, TN$ concur. Let $R$ be the point of concurrence. On the other hand, by Radical Axis on $(MQDG), (XQGN), (MFDNE)$, we have $MD, GQ, TN$ concur. Thus $R$ lies on $AH$ as well. We claim that $P, R, N$ are collinear.

Note $MO\parallel QHN\perp QA$, so $PM\perp QA$. Thus $PQ=PA$, so $P$ is the intersection of the tangents to $(AQFHGE)$ at $A, Q$. But recall $N$ was the intersection of the tangents at $E, F$. Thus by Pascal's on $AQHQAG$ we deduce $P, R, N$ are collinear, as desired. $\square$
This post has been edited 4 times. Last edited by DrMath, Jun 29, 2016, 5:15 PM
Z K Y
The post below has been deleted. Click to close.
This post has been deleted. Click here to see post.
MS_Kekas
275 posts
#14 • 4 Y
Y by Mindstormer, Didier, Adventure10, Mango247
Niche ne ponel no pooral
Z K Y
The post below has been deleted. Click to close.
This post has been deleted. Click here to see post.
v_Enhance
6882 posts
#15 • 8 Y
Y by niraekjs, smy2012, AlastorMoody, v4913, HamstPan38825, Adventure10, Mango247, Rounak_iitr
First wrote:
V_Enhance, how did you come up with this problem?
Well, the original proposal was to show that the nine-point circle and $(MBC)$ intersected on line $PN$. The points $G$ and $Q$ were added after the fact, to prevent people from using straight coordinates.

Here is also my original solution: Let $L$ be diametrically opposite $A$ on the circumcircle. Denote by $\triangle DEF$ the orthic triangle. Let $X = \overline{AH} \cap \overline{EF}$. Finally, let $T$ be the second intersection of $(MFDNE)$ and $(MBC)$.

We begin with a few easy observations. First, points $H$, $G$, $N$, $L$ are collinear and $\angle AGL = 90^\circ$. Also, $Q$ is the foot from $H$ to $\overline{AN}$. Consequently, lines $AG$, $EF$, $HQ$, $BC$, $TM$ concur at a point $R$ (radical axis). Moreover, we already know $\angle MTN = 90^\circ$. This implies $T$ lies on the circle with diameter $\overline{RN}$, which is exactly the circumcircle of $\triangle GQN$.

Note by Brokard's Theorem on $AFHE$, the point $X$ is the orthocenter of $\triangle MBC$. But $\angle MTN = 90^\circ$ already, and $N$ is the midpoint of $\overline{BC}$. Consequently, points $T$, $X$, $N$ are collinear.

Finally, we claim $P$, $X$, $N$ are collinear, which solves the problem. Note $P = \overline{GG} \cap \overline{AA}$. Set $K = \overline{HNL} \cap \overline{AP}$. Then by noting \[ -1 = (D,X;A,H) \overset{N}{=} (\infty, \overline{NX} \cap \overline{AK}; A, K) \]we see that $\overline{NX}$ bisects segment $\overline{AK}$, as desired. (A more projective finish is to show that $\overline{PXN}$ is the polar of $R$ to $\gamma$).
Attachments:
Z K Y
The post below has been deleted. Click to close.
This post has been deleted. Click here to see post.
Aiscrim
409 posts
#17 • 1 Y
Y by Adventure10
Let $A^\star$ be the antipode of $A$ in $(ABC)$ and let $QN\cap (ABC)=\{A,Q^\star\}$. It is well known that $A^\star$ is the reflection of $H$ over $N$. As $HQ\parallel Q^\star A^\star$ (they are both perpendicular to $AN$) and $NH=NA^\star$, we infer that $HQA^\star Q^\star$ is a parallelogram, so $NQ=NQ^\star$. Also note that $G-H-N$ are collinear.

Let $\{T\}=AG\cap HQ,\ \{R\}=AH\cap GQ$. Then $T$ lies on the polar of $P$ wrt $\gamma$, so $P$ lies on the polar of $T$ wrt to $\gamma$, i.e. $P\in NR$. Observe that by Power of Point we have
$$ NG\cdot NH=NA\cdot NQ=NA\cdot NQ^\star=NB\cdot NC$$
The inversion $\mathcal{I} (N,NB^2)$ swaps $G$ with $H$, $Q$ with $A$, and leaves $B$ and $C$ invariant. Let $\mathcal{I}(M)=M^\star$. Under inversion, the concurrency of $(NGQ),(MBC)$ and $PN$ is equivalent to the concurrency of $AH,(M^\star BC)$ and $NP$, i.e. to proving that $R \in (M^\star BC)$.

A little computation shows us that $BR\perp MC$, whence $R$ is the orthocenter of $\triangle{MBC}$, so $$\widehat{BRC}=180^\circ-\widehat{BMC}=\widehat{BM^\star C}\Rightarrow R \in (M^\star BC)$$
Z K Y
G
H
=
a